Last visit was: 26 Apr 2024, 01:39 It is currently 26 Apr 2024, 01:39

Close
GMAT Club Daily Prep
Thank you for using the timer - this advanced tool can estimate your performance and suggest more practice questions. We have subscribed you to Daily Prep Questions via email.

Customized
for You

we will pick new questions that match your level based on your Timer History

Track
Your Progress

every week, we’ll send you an estimated GMAT score based on your performance

Practice
Pays

we will pick new questions that match your level based on your Timer History
Not interested in getting valuable practice questions and articles delivered to your email? No problem, unsubscribe here.
Close
Request Expert Reply
Confirm Cancel
User avatar
Intern
Intern
Joined: 11 Apr 2008
Posts: 47
Own Kudos [?]: 202 [0]
Given Kudos: 0
Location: Chicago
Send PM
User avatar
Current Student
Joined: 12 Jul 2008
Posts: 366
Own Kudos [?]: 303 [0]
Given Kudos: 0
Concentration: Finance, Entrepreneurship, Health Care
Schools:Wharton
 Q50  V44
Send PM
User avatar
Intern
Intern
Joined: 11 Apr 2008
Posts: 47
Own Kudos [?]: 202 [0]
Given Kudos: 0
Location: Chicago
Send PM
User avatar
VP
VP
Joined: 29 Aug 2007
Posts: 1021
Own Kudos [?]: 1726 [0]
Given Kudos: 19
Send PM
Re: I'm having trouble with the rule behind this concept. This [#permalink]
brokerbevo wrote:
I'm having trouble with the rule behind this concept. This problem is a good example of my confusion:

If y is an integer, is y divisible by 3?
(1) y = 2x^3 + 9x^2 - 5x
(2) x is indivisible by 3

Please explain your rationale behind your answer.

Here is what I did:

(1) factor out an x --> x (2x^2 + 9x - 5)
factor the quadratic --> x (2x - 1) (x + 5)
so, y is the product of 3 integers and I wrote down that since 3 integers are multiplied together, it must be that it is divisible by 3. HOWEVER, I thought that this rule ONLY applies if the integers that are multiplied together are consecutive!!!

For example, x (x - 1) (x -4) is not divisible by 3 (even though it is the product of 3 numbers) because the parts are not consecutive.

Please explain...


you are almost there.

(1) y = 2x^3 + 9x^2 - 5x = x (2x^2 + 9x - 5) = x (2x - 1) (x + 5)

plug-in: any integer value for x in the above expression is divisible by 3.
if x = 2: x (2x - 1) (x + 5) = 2 (2x2 - 1) (2+5) = 2 x 3 x 7. suff.
if x = 4: x (2x - 1) (x + 5) = 4 (2x4 - 1) (4+5) = 4 x 7 x 9. suff.
if x = 5: x (2x - 1) (x + 5) = 5 (2x5 - 1) (5+5) = 4 x 9 x 10. suff.
if x = 7: x (2x - 1) (x + 5) = 7 (2x7 - 1) (7+5) = 7 x 13 x 12. suff.

i believe every integer value for x result y divisible by 3.

2: x = 3k is not sufficient.

So //A//
GMAT Tutor
Joined: 24 Jun 2008
Posts: 4128
Own Kudos [?]: 9245 [0]
Given Kudos: 91
 Q51  V47
Send PM
Re: I'm having trouble with the rule behind this concept. This [#permalink]
Expert Reply
brokerbevo wrote:
I'm having trouble with the rule behind this concept. This problem is a good example of my confusion:

If y is an integer, is y divisible by 3?
(1) y = 2x^3 + 9x^2 - 5x
(2) x is indivisible by 3

Please explain your rationale behind your answer.

Here is what I did:

(1) factor out an x --> x (2x^2 + 9x - 5)
factor the quadratic --> x (2x - 1) (x + 5)
so, y is the product of 3 integers and I wrote down that since 3 integers are multiplied together, it must be that it is divisible by 3. HOWEVER, I thought that this rule ONLY applies if the integers that are multiplied together are consecutive!!!

For example, x (x - 1) (x -4) is not divisible by 3 (even though it is the product of 3 numbers) because the parts are not consecutive.

Please explain...


Well, it doesn't *only* apply when the integers are consecutive. Say x is an integer. You know already that one of x, x+1 or x+2 is a multiple of 3. Well, that guarantees that one of, say, x, x+4, and x+8 is divisible by 3 (because if x+1 is, then x+1+3 = x+4 is, and if x+2 is, so is x+2 + 6 = x+8), and many other combinations besides.

You can apply that logic here if you want. We know that one of these numbers: x, x+1 or x+2 is a multiple of 3:

-If x is a multiple of 3, then x is a multiple of 3
-If x+1 is a multiple of 3, then 2x+2 is also a multiple of 3, and (subtract 3) 2x-1 is also a multiple of 3
-If x+2 is a multiple of 3, then x+2+3 = x+5 is also a multiple of 3.

So one of x, 2x-1 or x+5 will always be a multiple of 3, and since y = x (2x - 1) (x + 5), y must be a multiple of 3.

There are other ways to go about the question that might be preferable here (modular arithmetic, for example); I just wanted to show how you can extend the logic about 'consecutive numbers' to this situation.
User avatar
Director
Director
Joined: 17 Jun 2008
Posts: 614
Own Kudos [?]: 444 [0]
Given Kudos: 0
Send PM
Re: I'm having trouble with the rule behind this concept. This [#permalink]
But, the question stem does not specifically mention that x is an integer....hence I will go with C.
User avatar
VP
VP
Joined: 29 Aug 2007
Posts: 1021
Own Kudos [?]: 1726 [0]
Given Kudos: 19
Send PM
Re: I'm having trouble with the rule behind this concept. This [#permalink]
scthakur wrote:
But, the question stem does not specifically mention that x is an integer....hence I will go with C.


Do you have any non-integer value for x that yeilds an integer value for y?


any example?
User avatar
Director
Director
Joined: 30 Jun 2008
Posts: 538
Own Kudos [?]: 2730 [0]
Given Kudos: 1
Send PM
Re: I'm having trouble with the rule behind this concept. This [#permalink]
GMAT TIGER wrote:
brokerbevo wrote:
I'm having trouble with the rule behind this concept. This problem is a good example of my confusion:

If y is an integer, is y divisible by 3?
(1) y = 2x^3 + 9x^2 - 5x
(2) x is indivisible by 3

Please explain your rationale behind your answer.

Here is what I did:

(1) factor out an x --> x (2x^2 + 9x - 5)
factor the quadratic --> x (2x - 1) (x + 5)
so, y is the product of 3 integers and I wrote down that since 3 integers are multiplied together, it must be that it is divisible by 3. HOWEVER, I thought that this rule ONLY applies if the integers that are multiplied together are consecutive!!!

For example, x (x - 1) (x -4) is not divisible by 3 (even though it is the product of 3 numbers) because the parts are not consecutive.

Please explain...


you are almost there.

(1) y = 2x^3 + 9x^2 - 5x = x (2x^2 + 9x - 5) = x (2x - 1) (x + 5)

plug-in: any integer value for x in the above expression is divisible by 3.
if x = 2: x (2x - 1) (x + 5) = 2 (2x2 - 1) (2+5) = 2 x 3 x 7. suff.
if x = 4: x (2x - 1) (x + 5) = 4 (2x4 - 1) (4+5) = 4 x 7 x 9. suff.
if x = 5: x (2x - 1) (x + 5) = 5 (2x5 - 1) (5+5) = 4 x 9 x 10. suff.
if x = 7: x (2x - 1) (x + 5) = 7 (2x7 - 1) (7+5) = 7 x 13 x 12. suff.

i believe every integer value for x result y divisible by 3.

2: x = 3k is not sufficient.

So //A//



GMAT TIGER, why are we trying only integer values of x here ? The stem does not mention anything about x, it could be a fraction also.
User avatar
Director
Director
Joined: 30 Jun 2008
Posts: 538
Own Kudos [?]: 2730 [0]
Given Kudos: 1
Send PM
Re: I'm having trouble with the rule behind this concept. This [#permalink]
IanStewart wrote:

-If x is a multiple of 3, then x is a multiple of 3
-If x+1 is a multiple of 3, then 2x+2 is also a multiple of 3, and (subtract 3) 2x-1 is also a multiple of 3
-If x+2 is a multiple of 3, then x+2+3 = x+5 is also a multiple of 3.

So one of x, 2x-1 or x+5 will always be a multiple of 3, and since y = x (2x - 1) (x + 5), y must be a multiple of 3.

There are other ways to go about the question that might be preferable here (modular arithmetic, for example); I just wanted to show how you can extend the logic about 'consecutive numbers' to this situation.



If x is a multiple of 3, then x is a multiple of 3 . I guess there is a typo or am I missing the concept ?

Ian how do we apply modular arithmetic here ? Please guide.

Thanks
GMAT Tutor
Joined: 24 Jun 2008
Posts: 4128
Own Kudos [?]: 9245 [0]
Given Kudos: 91
 Q51  V47
Send PM
Re: I'm having trouble with the rule behind this concept. This [#permalink]
Expert Reply
amitdgr wrote:

If x is a multiple of 3, then x is a multiple of 3 . I guess there is a typo or am I missing the concept ?



No typo- I was just pointing out that one of x, x+5 or 2x-1 must be a multiple of 3, and if x is a multiple of 3, then it's just obvious that one of the three expressions is a multiple of 3.

I can explain how to approach this with modular arithmetic, but be aware that you won't need modular arithmetic on the GMAT. It would take me too long to explain the theory, so if you don't know modular arithmetic already, don't worry about it!

We want to know if

2x^3 + 9x^2 - 5x ~ 0 (mod 3)

We don't need to bother simplifying, but it makes things slightly easier: since 9 ~ 0 mod 3, and -5 ~ 1 mod 3, we just want to know if:

2x^3 + x ~ 0 (mod 3)

Now it's easy to check that if x ~ 0, 1 or 2 (mod 3), that 2x^3 + x is always congruent to 0 (mod 3), so it must be divisible by 3.
GMAT Tutor
Joined: 24 Jun 2008
Posts: 4128
Own Kudos [?]: 9245 [0]
Given Kudos: 91
 Q51  V47
Send PM
Re: I'm having trouble with the rule behind this concept. This [#permalink]
Expert Reply
GMAT TIGER wrote:
scthakur wrote:
But, the question stem does not specifically mention that x is an integer....hence I will go with C.


Do you have any non-integer value for x that yeilds an integer value for y?


any example?


y = 2x^3 + 9x^2 - 5x is a continuous function with an unrestricted range, so y can take on any value at all (graph it on the co-ordinate plane and you'll see why). I've ignored the possibility, in my posts above, that x is not an integer, because on the GMAT you would never see a question like this unless you were told in the question that x is an integer. I assumed there was a typo in the question, and that it was meant to begin "If x is an integer" rather than "If y is an integer". If the question doesn't mention this, it's really testing whether you know properties of continuous functions, something you learn in calculus, not in GMAT prep.

Still, if we accept the possibility that x is not an integer, the answer is not C, since Statement 2 is not at all helpful in this case; the answer would be E. But I'm sure the intended answer is A, and that the question intends for x to be an integer.



Archived Topic
Hi there,
This topic has been closed and archived due to inactivity or violation of community quality standards. No more replies are possible here.
Where to now? Join ongoing discussions on thousands of quality questions in our Quantitative Questions Forum
Still interested in this question? Check out the "Best Topics" block above for a better discussion on this exact question, as well as several more related questions.
Thank you for understanding, and happy exploring!
GMAT Club Bot
Re: I'm having trouble with the rule behind this concept. This [#permalink]
Moderator:
Senior Moderator - Masters Forum
3137 posts

Powered by phpBB © phpBB Group | Emoji artwork provided by EmojiOne